subject
Mathematics, 22.04.2020 01:36 anthonysf

Using strong induction (so, neither weak nor "weak++") on the number of matches in the pile, prove that if this number is of the form 4k + 1, for k ∈ N, then P2 always has a strategy to win (which they will follow, because they are smart). Otherwise, P1 always have a strategy to win (which they will also follow because they’re smart too)! To be clear, you will need to prove both facts with a single inductive proof! That is to say, you will need to prove that P2 can always win under the aforementioned condition, but whenever that condition is not met, you have to prove that it is now P1 that can always win!

ansver
Answers: 2

Other questions on the subject: Mathematics

image
Mathematics, 21.06.2019 18:30, Racc
Divide. write in simplest form 41/6 divided by 10
Answers: 1
image
Mathematics, 21.06.2019 18:40, sunshine52577oyeor9
20 points for the brainliest? drag each tile to the correct box. not all tiles will be used. arrange the steps to solve the equation . plz
Answers: 2
image
Mathematics, 22.06.2019 03:10, castilloitc1120
If the alternate hypothesis of an experiment is “the true mean height of the giraffes is more than 15 feet” what is the null hypothesis?
Answers: 1
image
Mathematics, 22.06.2019 06:00, kkmcfly
In the expansion of (2m-3n)^9 one of the terms contains m^3. determine the exponent of n in this term
Answers: 1
You know the right answer?
Using strong induction (so, neither weak nor "weak++") on the number of matches in the pile, prove t...

Questions in other subjects:

Konu
Mathematics, 17.11.2020 20:50
Konu
English, 17.11.2020 20:50
Konu
Mathematics, 17.11.2020 20:50